Keresés

Részletes keresés

mmormota Creative Commons License 2009.01.30 0 0 52595
"Ezt az erővonalak beszükülésére tudnám visszavezetni"

Nem jó szemlélet. A természet egy alapvető szimmetriatulajdonságát próbálnád "visszavezetni" más dolgokra, amik bonyolultabbak és amikből nem _következik_ ez a szimmetria, hanem azokra is _érvényes_.
Előzmény: Aurora11 (52584)
Mungo Creative Commons License 2009.01.30 0 0 52594

Amúgy van olyan kisérlet,hogy egymásra merőleges elektromos és mágneses térben Poynting-vektor nem nulla,folyik energiaáram.

 

Azért ezzel a "folyik energiaáram" dologgal nem árt óvatosan bánni.

Az elrendezés egyszerű, végy egy hengerkondenzátort, töltsd fel, helyezd olyan sztatikus mágnestébe, hogy a mágneses erővonalak legyenek párhuzamosak a hengerkondenzátorod szimmetriatengelyével.

A kondenzátor lemezei közötti tében a Pointing vektor nem nulla. De ezt az "energiaáramot" valamiért nem tudod megcsapolni. Ez nem kelt benned "gyanakvást"?

Előzmény: Aurora11 (52590)
NevemTeve Creative Commons License 2009.01.30 0 0 52593
Mindjárt kimutatja valaki, hogy a forgás inerciális mozgás... A centrifugális erő meg nyilván csak illúzió...
Mungo Creative Commons License 2009.01.30 0 0 52592

Az erővonalak beszükülése miatt fellép ...

 

Ezek az erővonaldolgok beszűkülése többnyire csak az agyi érelmeszesedésben szenvedő topikolómániások fejében fordul elő. (Pláne, ha elgurul a gyógyszerük.)

Ha te tényleg fizikusnak tanulsz, akkor inkább a tankönyvekkel próbálkozzál, mint ilyen kétesértékű helyen tájékozódni a világ dolgai felől. Ha már biztos talajon állsz, akkor könnyen szelektálni tudod a valódi értéket a zavaros elmék lázálmaitól.

Előzmény: Aurora11 (52589)
ivivan Creative Commons License 2009.01.30 0 0 52591
"Mennyivel rendkívűlibb a test forgása,mint lerövidülése?"
Mert ugyanis paradoxon az, hogy mindketten a másikat látjátok a saját tengelye körül forgónak. Könnyű ezt belátni: van két űrhajó, mingkettőben egy-egy megfigyelő. Mindkét megfigyelő kinéz az egyik ablakon és azt látja, hogy Ő maga nem forog (azaz mindig ugyanott látja a másik űrhajót), de a másik űrhajó forog.
Ez nyilvánvalóan ellentmondás: amikor az egyik megfigyelő szerint a másik épp úgy forog, hogy a másik oldalon néz ki, akkor nem is láthatná!

"A nézésünktől van hosszrövidülés?"
Van hosszrövidülés, de nem úgy, ahogy te képzeled, tehát nincs erőhatás: a rúd egyszerűen feszültségmentesen rövidebb, ahogyan az órás példámban is látszott...
Előzmény: Aurora11 (52587)
Aurora11 Creative Commons License 2009.01.30 0 0 52590

"Ezt levezetéssel, kísérleti megfigyeléssel alátámasztva is elő tudod adni, vagy csak a Gézoo-Ciprián páros értékes gondolatai okoztak ilyen mélyreható mentális traumát, melynek lilaszínű felhőcskéinek egyike a fenti idézet?  :o)"

 

Én szerintem rúdat sem látot senki sem lerövidülni,és nem hiszem hogy makroszkópikus méretű testet valaha is a fénysebesség közelébe tudnák gyorsítani.

 

Amúgy van olyan kisérlet,hogy egymásra merőleges elektromos és mágneses térben Poynting-vektor nem nulla,folyik energiaáram.És egy kondenzátorral a perdületet láthatóvá is lehet tenni.És a fényszórási effekktus is ismert,ami a terek beszükülésével ekvivalens folyamat.

Előzmény: Mungo (52588)
Aurora11 Creative Commons License 2009.01.30 0 0 52589

"Úgy általánosságban azért arról is kéne nyilatkozni, hogy például a nézésedtől hogyan kap esetleg perdületet a nézett rendszer?"

 

Az erővonalak beszükülése miatt fellép a Poynting-vektora,ami perdületet jelent.Ezt kell kikompenzálnia a rúd ellentétes irányú forgásának.

Előzmény: Mungo (52586)
Mungo Creative Commons License 2009.01.30 0 0 52588

Igen,az álló rendszerbeli megfigyelő pedig minket látna forogni.

 

Ezt levezetéssel, kísérleti megfigyeléssel alátámasztva is elő tudod adni, vagy csak a Gézoo-Ciprián páros értékes gondolatai okoztak ilyen mélyreható mentális traumát, melynek lilaszínű felhőcskéinek egyike a fenti idézet?  :o)

 

Mennyivel rendkívűlibb a test forgása,mint lerövidülése?

 

Csak annyival, hogy ilyen jelenséget, mármint az egymáshozképest nagy sebességgel mozgó inerciarendszerek "forgása" még semmilyen tapasztalat nem támasztja alá, míg a rövidülés mellett vannak megfigyelési eredmények. (Igaz közvetettek, de vannak.)

Előzmény: Aurora11 (52587)
Aurora11 Creative Commons License 2009.01.30 0 0 52587

Szia Mungo!

 

"Ha belegondolsz, hogy egy nagysebességű űrhajóból nézel egy álló rendszert, akkor azt is mondhatod, hogy te állsz és az általad nézett rendszer mozog."

Igen,az álló rendszerbeli megfigyelő pedig minket látna forogni.

 

"Ha belegondolsz, hogy egy nagysebességű űrhajóból nézel egy álló rendszert, akkor azt is mondhatod, hogy te állsz és az általad nézett rendszer mozog."

Mennyivel rendkívűlibb a test forgása,mint lerövidülése?A nézésünktől van hosszrövidülés?

Előzmény: Mungo (52586)
Mungo Creative Commons License 2009.01.30 0 0 52586

Tehát a hossz lerövidül,viszont a mozgásirány,mint tengely körül forog.

 

 

Szóval ezt még tényleg át kellene gondolnod.

Ha belegondolsz, hogy egy nagysebességű űrhajóból nézel egy álló rendszert, akkor azt is mondhatod, hogy te állsz és az általad nézett rendszer mozog.

Namármost akkor neked, vagy az általad nézett rendszernek kell forognia?

Úgy általánosságban azért arról is kéne nyilatkozni, hogy például a nézésedtől hogyan kap esetleg perdületet a nézett rendszer?

Előzmény: Aurora11 (52584)
ivivan Creative Commons License 2009.01.30 0 0 52585
Konkrétan ezt írtad: "v=0,8c X és X' távolsága 0,6cs szerinted mennyi ha nem 37 fok?"

Szöget mondjuk egy derékszögű háromszög alapján számolhatunk, ahogyan te is számoltad. De ehhez kell a derékszögű háromszög két befogója: az egyik a 0,6c távolság a két tengely között, de mi a másik? A sebesség miért lenne erre jó? És miért pont a forrás sebessége? Ha mozog ott egy kavics 0,3c-vel akkor azzal számolunk? Mi köze a beérkezett fotonnak a forrás sebességéhez?
Előzmény: Gézoo (52582)
Aurora11 Creative Commons License 2009.01.30 0 0 52584

Olyasmire gondoltam,hogy a fénysebesség és az idő alapján határozzuk meg a méteregységet.

 

De erről lemondtam,mert eszerint a fénysebesség a különböző sebességű IR-ekben eltérő lenne.Elismerem a hosszkontrakció valóságos hosszúságrövidülését.Ezt az erővonalak beszükülésére tudnám visszavezetni,csak ekkor fellépő Poynting-vektor perdületet hordoz,ezért a rúd el kezd forogni a mozgásirány mint tengely körül.

Tehát a hossz lerövidül,viszont a mozgásirány,mint tengely körül forog.

Előzmény: mmormota (52578)
Aurora11 Creative Commons License 2009.01.30 0 0 52583

"Induljunk ki az ismertből: egy adott  nyugalmi tömeg minden IR-ben ugyanazon nyugalmi tömeg. Azaz a vele egyenértékű energia szintén minden IR-ben azonos."

 

Igen,a tömeg mindenféle inerciarendszerből nézve invariáns.

 

"nyílván mondhatnánk, hogy csupán "a mérésnél alkalmazott mérték egységek nagysága változott".

  De ez nagyon téves következtetésekhez vezetne!

 

    Ugyanis a mértékegységeink sem változnak, semelyik IR-beli mércéinken sem."

De ha a mértékegységeik sem változik meg,és a nagysága sem,akkor semmilyen értelemben sincs hosszkontrakció?

 

 

Előzmény: Gézoo (52564)
Gézoo Creative Commons License 2009.01.30 0 0 52582

A detektorpár rendszerében a forrás sebessége v=0,8c.

 

  Mi mozogjon még 0,8c sebességgel? Mire gondolsz?

 

  A forráshoz rögzített koordináta rendszerre? Vagy mi másra?

Előzmény: ivivan (52580)
pint Creative Commons License 2009.01.30 0 0 52581
kéne már találni erre a jelenségre egy nevet. arra gondolok, hogy ismétlődő minta az, hogy egyes emberek fejében egy dolog két dologként jelenik meg, valószínűleg azért, mert ellentmondást érzékelnek az adott dolog természete és a fejükben levő józanparaszti modell között, és ezért a fogalmat kettéhasítják, az egyik fele "rendesen" működik, a másik fele meg "furán". a fogalom két képe a fejükben valahogy egymásra van montírozva, és mindig ott fed át, ahova éppen néznek.

példák:

a dolgok nem húzódnak össze, csak a méteregység

a látszat az, hogy a GSP holdak sietnek, de valójában nem (itt az egész világ duplikálva lett egy mérhetőre és egy valóságosra)

gézoo találta fel, hogy a fotonnak van sebességvektora és irányvektora (kis nyilak voltak a fotonokra rajzolva). a sebsségvektor olyan csúnya, hogy nézőpontfüggő, de az irányvektor meg szép, amilyennek gézoo szereti.

javasolt név: Oscar effektus. az Oscar című vígjátékra gondolok, amiben hősük ilyeneket mondott: tudtam, hogy a lányom, csak azt nem tudtam, hogy én vagyok az apja!


Előzmény: mmormota (52578)
ivivan Creative Commons License 2009.01.30 0 0 52580
Aha. És a detektor rendszerében mi a 0,8c? (Már persze a forrás sebességén kívül...)
Előzmény: Gézoo (52579)
Gézoo Creative Commons License 2009.01.30 0 0 52579
v=0,8c X és X' távolsága 0,6cs  szerinted mennyi ha nem 37 fok?
Előzmény: ivivan (52576)
mmormota Creative Commons License 2009.01.30 0 0 52578
"a hossz ténylegesen nem változik,csak az adott IR-ben a méteregység változik"

Ez sík hülyeség.
A hossz per definíció egy arány a referenciához képest.
A méteregység meg per definíció a referencia. Ennek következtében a méteregység hossza per definíció nem változhat meg, mert saját magával méred és mindig pontosan 1. :-)

Ha ezt máshogy látod, akkor nem úgy gondolsz a referenciára és a mérésre, ahogy kellene.
Feltételezel valami misztikus, ideális hosszt, ami azonban nem tudsz definiálni sem.
Előzmény: Aurora11 (52570)
egy mutáns Creative Commons License 2009.01.30 0 0 52577

Kedves cíprian,

azt hiszem, nem sikerült zöld ágra vergődni a vonat hosszának megmérése példájában, tudod, amikben a visszavert fényjelek idejét kellett felhasználni.

Lenne a példával kapcsolatosan egy kérdésem.

Tegyük fel, hogy jön egy vonat (aminek a nyugalmi hossza = l) a sínen a bakter felé ismert v sebességgel.

Amikor a vonat eleje odaér a bakterhez (t=0), a hátul utazó fékező kitesz egy csonagot a sínre.

Kérdés: milyen messze lesz a csomag a baktertől?

Pontosítás:

"amikor": a bakter órája szerinti t=0, illetve a fékező órája szerinti t=0 pillanatban teszi ki a csomnagot fékező.

Ha két megoldás lenne a kétféle "pontosítás" szerint, akkor tegyen ki két csomagot, és a két csomag helyét kérdezem.

1m

ivivan Creative Commons License 2009.01.30 0 0 52576
Hogy jött ki neked a 37 fok?
Előzmény: Gézoo (52574)
Gézoo Creative Commons License 2009.01.30 0 0 52575

Szia Kedves Kilroy!

 

  A forráspár  mozog a detektorpárok rendszerében, párhuzamosan az X' tengellyel,

 

  Y'=-0,6cs távolságban van a forráspár haladási tengelye a detektorok X' tengelyétől.

 

   Azaz a beérkezés 0;0 helyén a detektorpár síkjának felezőjére állított merőleges

egybevág a fény tényleges haladási irányával.

 

  Köznapi megfogalmazással a forrásból egy téglalap egyik csúcsán elindult fény,

az átlója mentén megérkezik a másik csúcson lévő detektorpába.

 

   Így a detektorpár rendszerében ismert a fényút hossza= átló hossza.

 

 A Minden rendszerben c sebességű fényt használva mércének, mivel a detektorpár rendszerében a fény 1c*1s utat tett meg, így kiszámolják, hogy ehhez 1 sec idő kellett a fénynek. 

   Azaz a beérkezés időpontjából levonva 1 sec-et az indulásra t=-1 s időpontot kapnak.

 

 

Előzmény: Kilroy (52572)
Gézoo Creative Commons License 2009.01.30 0 0 52574

Nyílván   fi=37+180=217 fokos a beérkezés szöge. 

 

    Ehhez nem kell konferencia.  Lemérik és kész.

 

    Egyszerűen leolvassák a detektorok síkja és az X' tengely által bezárt szöget egy szögmérőről.

 

   Így nyílván a 217 fok nem derékszög, sőt még csak 270 fok sem.. 217 fok.

 

Előzmény: ivivan (52573)
ivivan Creative Commons License 2009.01.30 0 0 52573
"Ez az induláskori hely a K' rendszerben -0,8cs;-0,6cs koordináta."

A detektor a beérkezés idejét tudja és a beérkezés szögét, de nyilván ha utána a megfigyelők mindkét rendszerben összejönnek a rendes évi megfigyelő konferencián, akkor ki tudják rakni a történéseket.

Egyébként a beérkezés szöge nyilván merőleges lesz a menetirányra, tehát már itt nincs igazad. Miből gondolod, hogy a beérkezés szöge nem merőleges?
Előzmény: Gézoo (52571)
Kilroy Creative Commons License 2009.01.30 0 0 52572
Így kizárólag a beérkezés helyét, idejét és a forrás induláskori irányát ismeri

a detektorpár.

 

Lehet, hogy nem jol ertem (sot valoszinuleg igy van), de szamomra itt valami illogikusnak tunik. Szerintem a forrasnak is csak a 'beerkezeskori' iranyat ismerheti a detektorpar, legalabbis a te logikadat kovetve (hiszen azt mondod, hogy csak a beerkezett fotonrol tudhat a detektor[par]).

Mert az en logikam szerint viszont a beerkezes ideje es a forras 'indulaskori' iranya pontosan a foton elindulasi helyet jelentene, amit ilyeten modon nem is kene feltetelezni, ahogy ezt ivivantol szamonkered.

De mindez csak laikus elmelkedes...

Előzmény: Gézoo (52571)
Gézoo Creative Commons License 2009.01.30 0 0 52571

Majdnem.

 

  Az a baj a gondolatmenetedben, hogy feltételezed egy fotonról, hogy honnan indul.

 

   Ne feltételezd!  Pontosan mérd meg!  Csak a beérkezett fotonról tud a detektorpár! 

 

   Az elindulásáról nem tudhat! Mert nincs a fénynél gyorsabb hirnök ami az elindulásról értesíthetné!

 

   Így kizárólag a beérkezés helyét, idejét és a forrás induláskori irányát ismeri

a detektorpár.

   Ez az induláskori hely a K' rendszerben  -0,8cs;-0,6cs koordináta.

 

   Kizárólag ezt tudja méréssel igazolni a detektorok rendszere.

 

   Így a K' rendszerben a fényút hossza:  -0,8cs;-0,6cs pont és 0;0 pont közötti 1 cs távolság.

 

   Azt, hogy K' rendszerhez hogyan közelített a fény, csak Einstein és a forrás K rendszerének megfigyelői tudhatták.

  A K' rendszerbeli megfigyelők a fény megérkezéséig nem tudtak még a létéről sem.

 

 

 

 

 

 

Előzmény: ivivan (52569)
Aurora11 Creative Commons License 2009.01.30 0 0 52570

Szia Mormota!

 

Ez a méteregységváltozás azt akarta jelenti,hogy a hossz ténylegesen nem változik,csak az adott IR-ben a méteregység változik(vagyis az ottani egységrendszerben a métert jelentő beosztás más,mint a mienkben),ez amiatt lép fel,mert a métert időegységgel definiáljuk:egy adott idő alatt a fény ekkora távolságot tesz meg.Csak az a baj,hogy ehez a modellhez az időegységnek is látszólagosnak kéne változnia.Csak az a baj,hogy az idő ténylegesen megváltozik.Emiatt,ha csak a méteregység változik meg,de az idő ténylegesen megváltozna,akkor a fény sebessége a különböző viszonylagos sebességű IR-ekben más-és más lenne.Viszont a fénysebesség minden IR-ben azonos.Ezért fel kell tételeznem,hogy a hosszrövidülés valóságos.Viszont fellép az a probléma,hogy a mozgásra merőleges hossz miért nem változik,miért nincs megvastagodás ebben az irányban?Ezt az elektromos erővonalak beszükülése magyarázza,szerintem.Csak ez maga után vonja,hogy erre merőleges irányban(mozgásirányban) a mágneses erővonalak is beszükülnek.Viszont akkor mivel egymásra merőleges elektromos és mágneses térerősség van,ezért fel kell lépnie a mozgásirány,mint tengely körül keringőmozgást végző Poynting-vektornak.És mivel a perdületmegmaradást meg kell követelnie,emiatt a rúdnak ugyanezen tengely körül a Poynting-vektorral ellentétes irányban forgómozgást kell végeznie.Ha ez a modell igaz,akkor a rúd ténylegesen lerövidül,de csak a mozgásirányban,de emellett a mozgásirány,mint tengely körül forgómozgást is végez.Ilyen forgásról még feladtaokban sohasem hallottam.

Előzmény: mmormota (52556)
ivivan Creative Commons License 2009.01.30 0 0 52569
Tehát K rendszerben t=0 időpontban a detektor (0,0.6) koordinátán van, míg t=1 időpontban a (0.8,0.6) koordinátán. Igaz?

A fényimpulzust t=0 időpontban az origóból indítja a forrás, ekkor a forrás rendszerében a fényút nyilvánvalóan 1c, ahogy eddig is mondtuk.

A detektor rendszerében a fényimpulzus a (0,-0.6) koordinátáról indul, érkezni pedig nyilvánvalóan a (0,0) koordinátára fog. A megtett fényút a detektor rendszerében egyértelműen 0,6c, ahogy eddig is mondtam...
Előzmény: Gézoo (52565)
Aurora11 Creative Commons License 2009.01.30 0 0 52568
Igazad van.Csak ha a Poynting vektor a mozgásirányú tengely körül végzi a forgását,így a rúdnak is ugyanezen tengely körül kéne forognia.Ez a z-írányú,a rajz síkjára merőleges irányú kiterjedéssel van kapcsolatban,ami ténylegesen nem változik,mert ez a rud mozgására merőleges irány.A Poynting vektornak múszály fellépnie,ha az erővonalak összeszükülnek.
Előzmény: cíprian (52555)
Aurora11 Creative Commons License 2009.01.30 0 0 52567

Szia Cíprian!

 

Az a baj,hogy otthon nincs meg nekem a könyv.De a könyvtárban megnézem.És jelentkezni fogok.

Előzmény: cíprian (52560)
Gézoo Creative Commons License 2009.01.30 0 0 52565

Szia Kedves Iván!

 

  Az ábrát a K rendszerben nyugvó megfigyeló t=0 időpontban látja úgy, hogy a v sebességgel +x irányban haladó K' rendszerben nyugvó detektorpárhoz rendelt X',Y' koordináta rendszerrel a detektorpár x=0;y=0,6cs helyen van,

 

    majd Y' berajzolása helyett vékony függőleges vonal jelzi Y' tengely helyét és ezzel a detektor helyét t=1 időpontban.

 

   A fényutak is fel vannak tüntetve 0;0 pont azaz a forrás és egyben a K rendszerbeli origó és a Szintén a K rendszerbeli   0,8cs; 0,6cs  pont között.

 

   Azaz mozog az ábrán a K' rendszer a detektorpárral, és a forráspár fényjelei..

 K' rendszer origója t=1 időpontban egybeesik K rendszer 0,8cs;0,6cs pontjával.

 

   Ezen felül az ábrán láthatók a detektor és forrás síkok vékony fekete 127 fokos hajlású vonallal jelölve.

 

 

Előzmény: ivivan (52543)

Ha kedveled azért, ha nem azért nyomj egy lájkot a Fórumért!